Difference between revisions of "2013 AIME I Problems/Problem 8"

(Added extra solution.)
Line 3: Line 3:
  
 
== Solution ==
 
== Solution ==
The domain of the arcsin function is <math>[-1, 1]</math>, so <math>-1 \le log_{m}(nx) \le 1</math>.
 
  
<math>\frac{1}{m} \le nx \le m</math>
+
We know that the domain of <math>\arcsin</math> is <math>[-1, 1]</math>, so <math>-1 \le \log_m nx \le 1</math>. Now we can apply the definition of logarithms:
 +
<cmath>m^{-1} = \frac1m \le nx \le m</cmath> <cmath>\implies \frac{1}{mn} \le x \le \frac{m}{n}</cmath>
 +
Since the domain of <math>f(x)</math> has length <math>\frac{1}{2013}</math>, we have that
 +
<cmath>\frac{m}{n} - \frac{1}{mn} = \frac{1}{2013}</cmath> <cmath>\implies \frac{m^2 - 1}{mn} = \frac{1}{2013}</cmath>
  
<math>\frac{1}{mn} \le x \le \frac{m}{n}</math>
+
A larger value of <math>m</math> will also result in a larger value of <math>n</math> since the numerator increases faster than the denominator, so we want to find the smallest value of <math>m</math> that also results in an integer value of <math>n</math>. The problem states that <math>m > 1</math>. Thus, first we try <math>m = 2</math>:
 +
<cmath>\frac{3}{2n} = \frac{1}{2013} \implies 2n = 3 \cdot 2013 \implies n \notin \mathbb{Z} </cmath> 
 +
Now, we try <math>m=3</math>:
 +
<cmath>\frac{8}{3n} = \frac{1}{2013} \implies 3n = 8 \cdot 2013 \implies n = 8 \cdot 671 = 5368</cmath>
 +
Since <math>m=3</math> is the smallest value of <math>m</math> that results in an integral <math>n</math> value, we have minimized <math>m+n</math>, which is <math>5368 + 3 = 5371 \equiv \boxed{371} \pmod{1000}</math>.
  
<math>\frac{m}{n} - \frac{1}{mn} = \frac{1}{2013}</math>
+
==Solution 2==
 +
We start with the same method as above. The domain of the arcsin function is <math>[-1, 1]</math>, so <math>-1 \le \log_{m}(nx) \le 1</math>.
  
<math>n = 2013m - \frac{2013}{m}</math>
+
<cmath>\frac{1}{m} \le nx \le m</cmath> <cmath>\frac{1}{mn} \le x \le \frac{m}{n}</cmath> <cmath>\frac{m}{n} - \frac{1}{mn} = \frac{1}{2013}</cmath> <cmath>n = 2013m - \frac{2013}{m}</cmath>
  
For <math>n</math> to be an integer, <math>m</math> must divide <math>2013</math>, and <math>m > 1</math>. To minimize <math>n</math>, <math>m</math> should be as small as possible because increasing <math>m</math> will decrease <math>\frac{2013}{m}</math> , the amount you are subtracting, and increase <math>2013m</math> , the amount you are adding; this also leads to a small <math>n</math> which clearly minimizes <math>m+n</math>.
+
For <math>n</math> to be an integer, <math>m</math> must divide <math>2013</math>, and <math>m > 1</math>. To minimize <math>n</math>, <math>m</math> should be as small as possible because increasing <math>m</math> will decrease <math>\frac{2013}{m}</math>, the amount you are subtracting, and increase <math>2013m</math>, the amount you are adding; this also leads to a small <math>n</math> which clearly minimizes <math>m+n</math>.
  
We let <math>m</math> equal 3, the smallest factor of <math>2013</math> that isn't <math>1</math>. Then we have <math>n = 2013*3 - \frac{2013}{3} = 6039 - 671 = 5368</math>
+
We let <math>m</math> equal <math>3</math>, the smallest factor of <math>2013</math> that isn't <math>1</math>. Then we have <math>n = 2013*3 - \frac{2013}{3} = 6039 - 671 = 5368</math>
  
 
<math>m + n = 5371</math>, so the answer is <math>\boxed{371}</math>.
 
<math>m + n = 5371</math>, so the answer is <math>\boxed{371}</math>.
 
  
 
== See also ==
 
== See also ==
 
{{AIME box|year=2013|n=I|num-b=7|num-a=9}}
 
{{AIME box|year=2013|n=I|num-b=7|num-a=9}}
 
{{MAA Notice}}
 
{{MAA Notice}}

Revision as of 13:39, 30 December 2013

Problem 8

The domain of the function $f(x) = \arcsin(\log_{m}(nx))$ is a closed interval of length $\frac{1}{2013}$ , where $m$ and $n$ are positive integers and $m>1$. Find the remainder when the smallest possible sum $m+n$ is divided by 1000.

Solution

We know that the domain of $\arcsin$ is $[-1, 1]$, so $-1 \le \log_m nx \le 1$. Now we can apply the definition of logarithms: \[m^{-1} = \frac1m \le nx \le m\] \[\implies \frac{1}{mn} \le x \le \frac{m}{n}\] Since the domain of $f(x)$ has length $\frac{1}{2013}$, we have that \[\frac{m}{n} - \frac{1}{mn} = \frac{1}{2013}\] \[\implies \frac{m^2 - 1}{mn} = \frac{1}{2013}\]

A larger value of $m$ will also result in a larger value of $n$ since the numerator increases faster than the denominator, so we want to find the smallest value of $m$ that also results in an integer value of $n$. The problem states that $m > 1$. Thus, first we try $m = 2$: \[\frac{3}{2n} = \frac{1}{2013} \implies 2n = 3 \cdot 2013 \implies n \notin \mathbb{Z}\] Now, we try $m=3$: \[\frac{8}{3n} = \frac{1}{2013} \implies 3n = 8 \cdot 2013 \implies n = 8 \cdot 671 = 5368\] Since $m=3$ is the smallest value of $m$ that results in an integral $n$ value, we have minimized $m+n$, which is $5368 + 3 = 5371 \equiv \boxed{371} \pmod{1000}$.

Solution 2

We start with the same method as above. The domain of the arcsin function is $[-1, 1]$, so $-1 \le \log_{m}(nx) \le 1$.

\[\frac{1}{m} \le nx \le m\] \[\frac{1}{mn} \le x \le \frac{m}{n}\] \[\frac{m}{n} - \frac{1}{mn} = \frac{1}{2013}\] \[n = 2013m - \frac{2013}{m}\]

For $n$ to be an integer, $m$ must divide $2013$, and $m > 1$. To minimize $n$, $m$ should be as small as possible because increasing $m$ will decrease $\frac{2013}{m}$, the amount you are subtracting, and increase $2013m$, the amount you are adding; this also leads to a small $n$ which clearly minimizes $m+n$.

We let $m$ equal $3$, the smallest factor of $2013$ that isn't $1$. Then we have $n = 2013*3 - \frac{2013}{3} = 6039 - 671 = 5368$

$m + n = 5371$, so the answer is $\boxed{371}$.

See also

2013 AIME I (ProblemsAnswer KeyResources)
Preceded by
Problem 7
Followed by
Problem 9
1 2 3 4 5 6 7 8 9 10 11 12 13 14 15
All AIME Problems and Solutions

The problems on this page are copyrighted by the Mathematical Association of America's American Mathematics Competitions. AMC logo.png